Invertible and Non-Invertible Systems

Поделиться
HTML-код
  • Опубликовано: 23 окт 2024

Комментарии • 38

  • @vincentjiang3590
    @vincentjiang3590 4 года назад +31

    sir, you have saved my digital system one year ago. now i am studying on signals and systems, you save me again!!! thank you very muc

  • @john93562
    @john93562 4 года назад +9

    we love you sir! this kind of education is nowhere to be found but here!

  • @rajatpandey496
    @rajatpandey496 7 лет назад +4

    Very helpful lectures.I am preparing for my mid-sem exams which will start from 12th Sep.Thanks Neso Academy!!! But when you will post the remaining part's videos?? Whether i should wait or i should study by myself?? becoz i have to complete my syllabus before 12th sep. Plz suggest me

  • @gmounika7777
    @gmounika7777 3 года назад +2

    Neso Academy its not a channel name its a brand😍.....

  • @DipakKumar-yl5vh
    @DipakKumar-yl5vh 6 лет назад +2

    Sir pls make videos on communication System.... Analog & Digital Both..ur way of teaching is very helpful in solving problems

  • @rajeevsingh2694
    @rajeevsingh2694 7 лет назад +2

    Excellent lecture series sir..... will u finish signal n system in september ??

  • @gedalaarthika2497
    @gedalaarthika2497 3 года назад

    Thankq so much sir....It really very helpful sir..🙏🙏

  • @aluualsaadi3769
    @aluualsaadi3769 6 лет назад +1

    Thanks Neso Academy

  • @prashanthiprashu4211
    @prashanthiprashu4211 5 лет назад +1

    Hello Sir, 1). y(n)=x(2n)
    2). y(n)=x(n)-x(n-1)
    How to do these problems whether it is invertible or not.?

    • @DSaiVarma
      @DSaiVarma 5 лет назад

      1) invertible
      2) Non-invertible

    • @SiddhiPuranik
      @SiddhiPuranik 10 месяцев назад

      ⁠@@DSaiVarmahi can u help me out
      Are these invertible?
      1. Y(t)= x(t)+x(t-10)
      2. Y(t) = x(t)+0.5x(t-10)

    • @Mansoor.BSCEF22
      @Mansoor.BSCEF22 6 месяцев назад

      First one is invertible
      Second one is non invertible

  • @AnshKaurms
    @AnshKaurms 6 месяцев назад

    sir, what is the answer of the homework question 2?

  • @sufianal-ethawi7406
    @sufianal-ethawi7406 6 лет назад

    Thank you

  • @sushmanaik2228
    @sushmanaik2228 3 года назад

    Sir,Why the gain is unity when invertible system cascaded with inverse system

  • @iamhere2125
    @iamhere2125 7 лет назад

    sir mujh LTI ka wait h...........
    apke signals n systems k saare videos dekhkr notes bnaaa chuka hu.......i m hmbly waiting fr LTI.....my exam is nearer....hope u ll get me n try to sort it out

    • @rashmikapur2801
      @rashmikapur2801 7 лет назад

      sir apne kaha tha ki aap gate or ies question ki playlist bhi upload kroge ??

    • @riteshmadishetty8450
      @riteshmadishetty8450 3 года назад +2

      bhayya if you don't mind can you please share with me the notes of these lectures to me? It would be very helpful to me as i didn't prepare any notes yet, thankyou: My Insta id: hielboy_oo7

    • @iamhere2125
      @iamhere2125 3 года назад

      ​@@riteshmadishetty8450Sorry!! I wish I could help you Ritesh. But I prepared those notes in 2018, then I was living near to IGI. Now location and everything changed. I don't have those materials, almost all the notes got spread among juniors.
      DO ONE THING, VISIT NOTESHUB( noteshub.co.in/Signals-and-Systems ) OR PASSHOJAO for QUESTIONS AND NOTES both.

    • @iamhere2125
      @iamhere2125 3 года назад

      @@riteshmadishetty8450Sorry!! I wish I could help you Ritesh. But I prepared those notes in 2018, then I was living near to IGI. Now location and everything changed. I don't have those materials, almost all the notes got spread among juniors.
      DO ONE THING, VISIT NOTESHUB( @t ) OR PASSHOJAO for QUESTIONS AND NOTES both.

  • @Anuparamkarad
    @Anuparamkarad 5 лет назад +1

    In the second problem if x(t)=-6 y(t)=4; if x(t)=2 y(t)=4 how it is invertible

    • @zw9423
      @zw9423 5 лет назад +4

      it's not 4 but -4 when x(t) = -6

  • @debabrata2137
    @debabrata2137 7 лет назад

    How sint*del(t) = 0 , at t=0. At t = 0 , del(t) is infinity

  • @Oktheorignal
    @Oktheorignal 7 лет назад +1

    I found a mistake. At 06:30 Example 2 is NON Invertible. Check x(t) =0 and x(t)=-4. Both have the same O/P which is 2. So It is Non invertible. Please confirm Neso Academy

    • @multi-ytc1659
      @multi-ytc1659 7 лет назад +1

      Omar Kiyani by putting x(t)=-4 we get y(t)=-2, not 2 ... so it's invertible system

    • @Oktheorignal
      @Oktheorignal 7 лет назад

      Multi - YTC Sorry! I was binge watching for an exam and I miscalculated! Thanks

    • @rahulsinghthakur420
      @rahulsinghthakur420 5 лет назад

      @@Oktheorignal and if we keep -2 then output is zero so it is a non invertible system

    • @Vishwa2757
      @Vishwa2757 2 года назад

      @@rahulsinghthakur420 only for -2 it gives zero. for other values it won't give zero. So it is invertible system.

  • @rishikapadia7989
    @rishikapadia7989 3 года назад

    how to make non invertible system to an
    invertible system?

  • @priyanshupatel9521
    @priyanshupatel9521 5 лет назад

    X(t)+2 is non invertible..sir ji..because for a non zero input -2 output is zero..and we can't get back our input from zero

    • @youtubeonly7271
      @youtubeonly7271 5 лет назад +2

      chutiya hai kya

    • @Blue_Pumpkin
      @Blue_Pumpkin 5 лет назад

      @@youtubeonly7271 😂😂

    • @sushmanaik2228
      @sushmanaik2228 3 года назад

      The above question satisfies one to one mapping so,it is invertible.for input values there are different output values. I don't understand " we can't get back our input from zero?"

  • @priyatiwari6331
    @priyatiwari6331 6 лет назад

    Sir jaise first question mai y(t) x=xsqure t hai usme usme hum aur koi Vale rakhoge to kuch aur and ayga too hum kaise samjh ge

  • @ArjunSharma-el2tf
    @ArjunSharma-el2tf 5 лет назад

    what about
    y(t)=x(t-4)
    invertible or not ?

  • @RamanujanambiK
    @RamanujanambiK 2 месяца назад

    What is casecade

    • @vandanashivasaisurya3747
      @vandanashivasaisurya3747 18 дней назад

      Output of invertible system is giving as input to inverse system is known as cascade